From a78e5c604a6f2a37c955a1498970578e373d682c Mon Sep 17 00:00:00 2001 From: "W. Trevor King" Date: Tue, 25 Aug 2009 00:26:57 -0400 Subject: [PATCH] Completed rec9 solutions --- .../Young_and_Freedman_12/problem28.12.tex | 68 ++++++++- .../Young_and_Freedman_12/problem28.18.tex | 35 +++++ .../Young_and_Freedman_12/problem28.23.tex | 133 +++++++++++++++++ .../Young_and_Freedman_12/problem28.30.tex | 47 ++++++ .../Young_and_Freedman_12/problem28.60.tex | 136 ++++++++++++++++++ .../Young_and_Freedman_12/problem28.62.tex | 86 +++++++++++ 6 files changed, 498 insertions(+), 7 deletions(-) diff --git a/latex/problems/Young_and_Freedman_12/problem28.12.tex b/latex/problems/Young_and_Freedman_12/problem28.12.tex index 4b0ce19..30df69a 100644 --- a/latex/problems/Young_and_Freedman_12/problem28.12.tex +++ b/latex/problems/Young_and_Freedman_12/problem28.12.tex @@ -1,25 +1,54 @@ \newcommand{\dB}{d\vect{B}} \newcommand{\dl}{d\vect{l}} -\newcommand{\rhat}{\hat{r}} \begin{problem*}{28.12} - +Two parallel wires are $5.00\U{cm}$ apart and carry currents in +opposite directions, as shown in Fig.~28.37. Find the magnitude and +direction of the magnetic field at point $P$ due to the two +$1.50\U{mm}$ segments of wire that are opposite each other and +$8.00\U{cm}$ from point $P$. \begin{center} \begin{asy} import Mechanics; import ElectroMag; +import Circ; -real u = 0.1cm; +real u = 0.6cm; real Ysep = 5u; real hypot = 8u; real Xsep = sqrt(hypot**2 - (Ysep/2)**2); real Xslush = 1u; +real Xseg = 1u; + +Distance dtop = Distance((-Xsep,Ysep/2), (0,0), "$8.00\U{cm}$"); +Distance dbot = Distance((-Xsep,-Ysep/2), (0,0), "$8.00\U{cm}$"); + +dtop.draw(labeloffset=8pt); +dbot.draw(labeloffset=-8pt); +dot("P", (0,0)); + + +Wire wbot_seg = Wire((-Xsep-Xseg/2, -Ysep/2), (-Xsep+Xseg/2, -Ysep/2), red); +Wire wbot = Wire((-Xsep-Xslush, -Ysep/2), (Xslush,-Ysep/2)); -Wire wtop_seg = Wire( -Wire wtop = Wire((), (Xslush,Ysep/2)); +wbot.draw(); +wbot_seg.draw(); +TwoTerminal Ibot = current((0,-Ysep/2), 180, "$24.0\U{A}$"); +Distance dSegbot = Distance((-Xsep-Xseg/2,-Ysep/2), (-Xsep+Xseg/2,-Ysep/2), + offset=2mm, "$1.50\U{mm}$"); +dSegbot.draw(labeloffset=-8pt); + + +Wire wtop_seg = Wire((-Xsep-Xseg/2, Ysep/2), (-Xsep+Xseg/2, Ysep/2), red); +Wire wtop = Wire((-Xsep-Xslush, Ysep/2), (Xslush,Ysep/2)); wtop.draw(); wtop_seg.draw(); +TwoTerminal Itop = current((Ibot.end.x,Ysep/2), "$12.0\U{A}$"); +Distance dSegtop = Distance((-Xsep-Xseg/2,Ysep/2), (-Xsep+Xseg/2,Ysep/2), + offset=-2mm, "$1.50\U{mm}$"); +dSegtop.draw(labeloffset=8pt); + \end{asy} \end{center} \end{problem*} @@ -36,6 +65,32 @@ on both segments. All of the components are given except for $\dl\times\rhat$. Drawing a picture for the top segment \begin{center} \begin{asy} +import Mechanics; +import Circ; + +real u = 0.6cm; +real Ysep = 5u; +real hypot = 8u; +real Xsep = sqrt(hypot**2 - (Ysep/2)**2); +real Xslush = 1u; +real Xseg = 1u; + +draw((-Xsep,Ysep/2)--(0,0)--(-Xsep,0)--cycle, dashed); +label("$8.00$", (-Xsep/2,Ysep/4), NE); +label("$2.50$", (-Xsep,Ysep/4), W); + +dot("P", (0,0)); + +Angle theta = Angle((0,Ysep/2), (-Xsep,Ysep/2), (0,0), 10mm, "$\theta$"); +theta.draw(); +Angle theta2 = Angle((-Xsep,Ysep/2), (0,0), (-Xsep,0), 10mm, "$\theta$"); +theta2.draw(); + +Vector dL = Vector((-Xsep, Ysep/2), mag=3u, "$\dl$"); +Vector rhat = Vector((-Xsep, Ysep/2), mag=3u, dir=degrees((Xsep,-Ysep/2)), + "$\rhat$"); +dL.draw(); +rhat.draw(); \end{asy} \end{center} Using our knowledge of cross products and trigonometry @@ -43,7 +98,6 @@ Using our knowledge of cross products and trigonometry |\dl\times\rhat|=|\dl|\cdot|\rhat|\cdot\sin\theta = 1.50\U{mm}\cdot1\cdot\sin\theta = 1.50\U{mm}\cdot\frac{2.50\U{cm}}{8.00\U{cm}} - = 1.50\U{mm}\cdot\frac{2.50\U{cm}}{8.00\U{cm}} = 4.6875\E{-4}\U{m} \;. \end{equation} It's also pretty clear that this cross product will have the same @@ -53,7 +107,7 @@ work that out in detail if you didn't notice right off. The net magnetic field $B_p$ is then \begin{equation} B_p = B_{pt} + B_{pb} - = \frac{$\mu_0$}{4\pi}\cdot\frac{4.6875\E{-4}\U{m}}{(8.00\E{-2}\U{m})^2} + = \frac{\mu_0}{4\pi}\cdot\frac{4.6875\E{-4}\U{m}}{(8.00\E{-2}\U{m})^2} \cdot(12.0\U{A}+24.0\U{A}) = \ans{264\U{nT}} \;. \end{equation} diff --git a/latex/problems/Young_and_Freedman_12/problem28.18.tex b/latex/problems/Young_and_Freedman_12/problem28.18.tex index b585771..0fc786a 100644 --- a/latex/problems/Young_and_Freedman_12/problem28.18.tex +++ b/latex/problems/Young_and_Freedman_12/problem28.18.tex @@ -1,5 +1,40 @@ \begin{problem*}{28.18} +Two long, straight wires, one above the other, are seperated by a +distance $2a$ and are parallel to the $x$-axis. Let the $+y$-axis be +in the plane of the wires in the direction from the lower wire to the +upper wire. Each wire carries current $I$ in the $+x$-direction. +What are the magnitude and direction of the net magnetif field of the +two wires at a point in the plane of the wires \Part{a} midway between +them; \Part{b} at a distance $a$ above the upper wire; \Part{c} at a +distance $a$ below the lower wire? \end{problem*} \begin{solution} +\Part{a} +Using the right hand rules for magnetic field from a wire, we see that +the upper wire will create a magnetic field into the page while the +lower wire will create a magnetic field out of the page. The +magnitude of the field from a single wire is given by +\begin{equation} + B = \frac{\mu_0 I}{2\pi r} \;, +\end{equation} +with the same current $I$ and distance $r=a$ for both wires. +Therefore the net magnetic field is \ans{zero}. + +\Part{b} +Both wires create a magnetic field out of the page. The magnitude of +the toal field will be +\begin{equation} + B = \frac{\mu_0 I}{2\pi a} + \frac{\mu_0 I}{2\pi (3a)} + = \ans{\frac{2 \mu_0 I}{3\pi a}} \;. +\end{equation} + +\Part{c} +Both wires create a magnetic field into the page. The magnitude of +the toal field will be +\begin{equation} + B = \frac{\mu_0 I}{2\pi (3a)} + \frac{\mu_0 I}{2\pi a} + = \ans{\frac{2 \mu_0 I}{3\pi a}} \;, +\end{equation} +the same as the magnitude for \Part{b}. \end{solution} diff --git a/latex/problems/Young_and_Freedman_12/problem28.23.tex b/latex/problems/Young_and_Freedman_12/problem28.23.tex index e844ac4..cb59801 100644 --- a/latex/problems/Young_and_Freedman_12/problem28.23.tex +++ b/latex/problems/Young_and_Freedman_12/problem28.23.tex @@ -1,5 +1,138 @@ \begin{problem*}{28.23} +Four long, parallel power lines each carry $100\U{A}$ currents. A +cross-sectional diagram of these lines if a square, $20.0\U{cm}$ on +each side. For each of the three cases shown in Fig.~28.41, calculate +the magnetic field at the center of the square. \end{problem*} +\begin{nosolution} +\begin{center} +\begin{asy} +import Mechanics; + +real u = 1cm; +real a = 1u; +real dx = 3u; +pen ipen = red+blue; + +real x = 0; +real d = a/2; + +Vector Vs[]; + +label("(a)", (x,-d), S); +Vs.push(Vector((x-d,-d), phi=-90, ipen)); +Vs.push(Vector((x+d,-d), phi=-90, ipen)); +Vs.push(Vector((x+d, d), phi=-90, ipen)); +Vs.push(Vector((x-d, d), phi=-90, ipen)); +x += dx; + +label("(b)", (x,-d), S); +Vs.push(Vector((x-d,-d), phi= 90, ipen)); +Vs.push(Vector((x+d,-d), phi=-90, ipen)); +Vs.push(Vector((x+d, d), phi= 90, ipen)); +Vs.push(Vector((x-d, d), phi=-90, ipen)); +x += dx; + +label("(c)", (x,-d), S); +Vs.push(Vector((x-d,-d), phi= 90, ipen)); +Vs.push(Vector((x+d,-d), phi= 90, ipen)); +Vs.push(Vector((x+d, d), phi=-90, ipen)); +Vs.push(Vector((x-d, d), phi=-90, ipen)); + +for (int i=0; i